Calculating Volume of Rotated Region in Quadrant 1

  • Thread starter Thread starter Aerosion
  • Start date Start date
  • Tags Tags
    Volume
Click For Summary
The discussion focuses on calculating the volume of a solid formed by rotating the region in the first quadrant enclosed by the curves y = x^2 and y = 2x around the x-axis. The initial approach involved solving for y and setting up an integral, but it was identified that solving for x was unnecessary for this problem. The correct formula for the volume of revolution was provided, emphasizing the need to integrate the difference of the squares of the two functions. A visual sketch of the curves was also recommended to aid understanding. Ultimately, the participant confirmed they were able to arrive at the correct answer after clarifying the method.
Aerosion
Messages
52
Reaction score
0

Homework Statement



Find the volume of the solid formed by rotating the region inside the first quadrant enclosed by
y = x^2
y = 2x

Homework Equations


The Attempt at a Solution



Okay, so I first solved both equations for y, which gave me x=radical(y) and x=y/2. Then I graphed both of them, found that the radical one was above the y/2, so I made the equation pi*(radical(y)^2 - (y/2)^2. I then made that equation a definite integral with lower limit 0 and upper limit 2.

Of course, it turned out wrong (I'd use Latex to make it look nice, but it's not coming out very well right now and I don't have the patience). Suffice it to say, I did something wrong somewhere.
 
Last edited:
Physics news on Phys.org
About which line are you rotating the region about, the x axis?
 
Yes yes, the x axis. Sorry.
 
Then there is no need to solve for x. The volume of revolution about the a axis of a region bounded by two functions, f(x) and g(x) and the lines x=a and x=b is given by;

V=\pi\int_a^b{\left|\left[f(x)\right]^2-\left[g(x)\right]^2\right|}dx

It my also be a good idea to sketch the two curves to get a visual idea of what your are actually doing.
 
Last edited:
Ah okay, now I'm getting the right answer. Thanks.
 
Aerosion said:
Ah okay, now I'm getting the right answer. Thanks.
No worries :biggrin:
 
Question: A clock's minute hand has length 4 and its hour hand has length 3. What is the distance between the tips at the moment when it is increasing most rapidly?(Putnam Exam Question) Answer: Making assumption that both the hands moves at constant angular velocities, the answer is ## \sqrt{7} .## But don't you think this assumption is somewhat doubtful and wrong?

Similar threads

  • · Replies 1 ·
Replies
1
Views
2K
  • · Replies 3 ·
Replies
3
Views
3K
Replies
2
Views
1K
Replies
4
Views
2K
  • · Replies 4 ·
Replies
4
Views
2K
Replies
2
Views
2K
  • · Replies 27 ·
Replies
27
Views
3K
Replies
3
Views
2K
Replies
5
Views
2K
Replies
2
Views
1K